Derivative of an accumulation function.

  • Thread starter Thread starter InaudibleTree
  • Start date Start date
  • Tags Tags
    Derivative Function
Click For Summary
The discussion centers on the differentiation of an accumulation function defined as F(x) = ∫[π to ln(x)] cos(e^t) dt. The user initially applies a theorem for differentiation of integrals but arrives at an incorrect conclusion, suggesting F'(x) = cos(x). However, the correct approach involves recognizing that the upper limit of integration is a function of x, requiring the application of the Leibniz integral rule. This rule states that d/dx[∫[a to g(x)] f(t) dt] = f(g(x)) g'(x), leading to the correct answer of F'(x) = cos(ln(x)) * (1/x). Understanding this differentiation technique is crucial for solving similar problems accurately.
InaudibleTree
Messages
9
Reaction score
0

Homework Statement



F(x) = \int^{ln(x)}_{\pi}cos(e^t)\,dt

Homework Equations


The Attempt at a Solution



Following from a theorem given in the text I am using:

If f is continuous on an open interval I containing a, then, for every x in the interval,
d/dx[\int^x_af(t)\,dt] = f(x)

I thought it would be as simple as

F'(x) = d/dx[sin(e^{ln(x)})] = d/dx[sin(x)] = cos(x)

But according to the text the answer is cos(x)/x

So what am i doing wrong?
 
Physics news on Phys.org
InaudibleTree said:

Homework Statement



F(x) = \int^{ln(x)}_{\pi}cos(e^t)\,dt

Homework Equations


The Attempt at a Solution



Following from a theorem given in the text I am using:

If f is continuous on an open interval I containing a, then, for every x in the interval,
d/dx[\int^x_af(t)\,dt] = f(x)

I thought it would be as simple as

F'(x) = d/dx[sin(e^{ln(x)})] = d/dx[sin(x)] = cos(x)

But according to the text the answer is cos(x)/x

So what am i doing wrong?

It's not quite that simple if your limit isn't simply x. d/dx[\int^{g(x)}_af(t)\,dt] = f(g(x))g'(x). In general you have to use http://en.wikipedia.org/wiki/Leibniz_integral_rule
 
Question: A clock's minute hand has length 4 and its hour hand has length 3. What is the distance between the tips at the moment when it is increasing most rapidly?(Putnam Exam Question) Answer: Making assumption that both the hands moves at constant angular velocities, the answer is ## \sqrt{7} .## But don't you think this assumption is somewhat doubtful and wrong?

Similar threads

  • · Replies 15 ·
Replies
15
Views
2K
  • · Replies 5 ·
Replies
5
Views
2K
  • · Replies 14 ·
Replies
14
Views
2K
Replies
3
Views
2K
  • · Replies 105 ·
4
Replies
105
Views
6K
Replies
3
Views
2K
Replies
2
Views
2K
  • · Replies 23 ·
Replies
23
Views
2K
  • · Replies 3 ·
Replies
3
Views
2K
  • · Replies 11 ·
Replies
11
Views
3K